PrepTest 61 LSAT Logic Games Solutions #2

LSAT Blog PrepTest 61 Game 2 SolutionThis LSAT Blog post covers the second Logic Game of the October 2010 LSAT (PrepTest 61).

Don't look at these explanations until you've taken PrepTest 61 as a full-length timed exam.

Also see:

PrepTest 61 (October 2010 LSAT), Game 1 Explanation
PrepTest 61 (October 2010 LSAT), Game 3 Explanation
PrepTest 61 (October 2010 LSAT), Game 4 Explanation

Explanations for Recent LSAT Logic Games

***
This game concerns six ancient artifacts: Figurine, Headdress, Jar, Necklace, Plaque and Tureen being placed in order from oldest to newest (most recent).

The rules in this game are primarily relational, meaning they require that one variable occurs before or after another, so we're dealing with a Pure Sequencing game.

In these games, I like to build the diagram step-by-step.

In this particular game, I diagram the first rule, then connect it with the second, to form:
LSAT Blog PrepTest 61 Game 2 Basic Setup







Whatever's to the left of those lines is earlier than whatever's to the right.


Now, the final rule is a bit tricky. It sets up 2 main possibilities based upon whether P is older than both (to the left of both) or younger than both (to the right of both). Here are both possibilities for this 3rd rule:

LSAT Blog PrepTest 61 Game 2 Third Rule





Now, the hard part is connecting this rule to what we already have, since N and H are spaced far-apart in our connection of the 1st and 2nd rules. There are a number of ways to connect everything together, none of which are incredibly pretty or easy to create, but you'll want to connect the rules in order to make the greatest possible number of inferences.

Here's the connection of the 1st and 2nd rules with the possibility that P occurs before (is older than) both H and N:
LSAT Blog PrepTest 61 Game 2 Possibility 1










You'll notice that I flipped around N and H from what I had previously, and I also flipped around H and J - this was just to bring things together in order to more easily connect them. Whether a variable is higher or lower than another doesn't matter. What does matter is that the relationships are made explicit.

Ideally, we wouldn't have H in the middle of things, but there's not really a way around that.

Here's the alternative possibility, where the connection of the 1st and 2nd rules is combined with the possibility that P occurs after (is younger than) both H and N:

LSAT Blog PrepTest 61 Game 2 Possibility 2











Again, we've needed to do some top-bottom flipping to connect things more easily.

Again, it would've been nice if there weren't a variable in the middle (in this case, N), but there's not really a way around that.

Here are both possibilities, side by side:

LSAT Blog PrepTest 61 Logic Game Solutions Both Possibilities












Now, on to the questions.

Question 6:

Each of the incorrect choices will violate at least one rule.

Choice A happens to fit Possibility #2 and just jumped out at me, so I'd pick it and move on.

However, I'll explain why the others are invalid scenarios:

Choices B and E don't have both N and J before T (they have N after T). Eliminated.

Choice C doesn't have P either before both H and N or after both H and N. It has P between them, which isn't allowed. Eliminated.

Choice D doesn't have F before both J and H (in this choice, J is before F). Eliminated.


Question 7:
In possibility #1, either P or F could be 1st. In possibility #2, either F or N could be 1st. Therefore, generally speaking, any of P, F, or N could be 1st, and choice C is our answer.


Question 8:
We can automatically eliminate choice D. Because N was 4th in the correct answer to Question 6, we know it's possible for N to be 4th.

Now, if something can't go 4th, and we have only 6 variables, then we want to look at something that usually has to go pretty early. The one variable that could consistently be 1st (in both possibilities) is F.

Regardless of which possibility we're in, F always has at least 3 variables that must go after it. In Possibility #1, H, J, and T always have to go after F. In Possibility #2, H, T, P, and J must all go after F. Since there are only 6 artifacts total, F can't ever go 4th or later, so Choice A is our answer.


Question 9:
If F is 3rd, we must be in Possibility #1. Possibility #2 has at least 4 artifacts going after F, so F wouldn't be able to be 3rd in that possibility.

Within Possibility #1, if F is 3rd, H, J, and T must go after it on 4, 5, and 6, with J before T. P must go before N within the 1st 2 slots. This gives us:
LSAT Blog PrepTest 61 Logic Game Solutions Question 9




There are 3 different possible orderings based upon the placement of H, but we're concerned here with what is 2nd, and we know it must be N, so Choice C is our answer.


Question 10:
If P is 1st, we must be within Possibility #1.

If it's 1st, it's before all other variables, of course. In Possibility #1, it's already before every single variable except F. A new diagram representing this limitation will bring us to the answer:

LSAT Blog PrepTest 61 Logic Game Solutions  Question 10






I've flipped J and H around in order to connect J to T more smoothly.

Now, the question asks us how many could be 2nd. Looking at the new diagram, we can see that both N and F could be the next artifacts after P, but no other artifacts could come immediately after P. As such, Choice B (two) is our answer.


Question 11:

This is a rule substitution question. They're removing the 2nd rule that N and J are required to come before T, and asking us which choice, if true, would put things back as they were.

Well, removing that rule simply leaves us with F coming before H and J, as well as the rule that P is either before both N and H or it's after both N and H.

The correct answer will have to bring us back to our two main possibilities from before:
LSAT Blog PrepTest 61 Logic Game Solutions - Both Possibilities











As such, it's going to involve T is some way, and it's going to involve forcing T to go late (towards the end), since T is late in both diagrams. Now, all 5 choices involve T, of course, which makes things a bit more difficult.

Choice A creates the ordering "F - T - H", but we didn't have that requirement in both possibilities. In fact, we didn't have it in either. Eliminated.

Choice B places both F and N before T. While that was certainly true in both possibilities, this does not return things to normal because it doesn't put J before T. Eliminated.

Choice C says that either both N and J are before T, or both N and J are after T. However, we need both to be before T. Having both after T is not an option given the original possibilities. Eliminated.

Choice D requires that everything except H and P go before T. In other words, F, J, N would all have to go before T. Maybe H and P would also, maybe they wouldn't. However, this puts things back as they were before, by imposing that N and J go before T (since F is already before J, it must always go before T as well). Choice D is our answer.

I'll discuss Choice E anyway.

Choice E requires that P is either before both N and T or that it's after both N and T. In Possibility #1, P was before both N and T. However, in Possibility #2, we could've had the general ordering N - P - T. As such, this new constraint violates what was previously possible. Furthermore, this new constraint does not require both N and J to go before T, so it's not limiting things in the same way they were limited before. Eliminated.

Photo by flydime



1 comment:

  1. I found this blog after a long time which is really helpful to let understand different approaches. I am going to adopt these new point to my career and thankful for this help.
    poker news

    ReplyDelete